Putnam 2015
by BOGTRO, Dec 6, 2015, 10:16 PM
Well, my first experience with the Putnam contest was yesterday, and it was definitely an interesting experience. Beforehand, I didn't really have much idea what to expect; I'd done some "practice" problems with other people, but messing around with problems in a group is very different from working on them individually during a contest. What I did know is that the prerequisite knowledge base would presumably be higher than for high school contests -- in particular, calc and lin alg would probably be making appearances.
Unfortunately, despite 18.701 I'm apparently not very comfortable with lin alg, so I got absolutely nowhere on the one problem that used it. In my defense, this was at least partially because the entire B set was evil and I didn't even read B3 until a couple hours into the test; sadly, this also caused my proof-writing to be... somewhat lacking, so I might end up with three 1s on the three problems I solved on B (darn).
Anyway, these are my solutions, attempting to reconstruct exactly what I wrote on the test. After section A I was very confident since I was pretty sure I had four solid solutions (hopefully this is accurate), so I figured as long as I didn't completely bomb section B I should be in a solid spot to get HM. Unfortunately part A used up all the NT
A1
A2
A3
A4
Section B was much more stressful; I'd been actively hoping there wouldn't be too much calc, but B1 was some annoying diffeq that I didn't really have any good ideas how to approach (other than the coefficients suspiciously looking like
). At least it was a #1 so I could get through it quickly and move on to more interesting problems... or so my reasoning went until half an hour later I had zero progress. So I tried B2 for a while, but got absolutely nowhere, and B3 looked hard, so I floated aimlessly between B1 and B2 for a while while getting more and more frustrated.
Eventually I finished B2 first, using an induction that I wasn't (and still aren't) fully confident in, but I definitely solved the problem so hopefully it'll get a 10- grade. Finally, with about half an hour left, I finally came up with the insight as to how B1 would die, so I finished that with about 20 minutes left. Then I looked at B4, which was definitely the easiest problem of the set (maybe I should have looked at this before...), but time pressure made me butcher the computation at the end, and apparently Putnam is very unforgiving towards irrelevant mistakes so that might get a 1 T.T
B1
Motivation
B2
B4
Better finish
Unfortunately, despite 18.701 I'm apparently not very comfortable with lin alg, so I got absolutely nowhere on the one problem that used it. In my defense, this was at least partially because the entire B set was evil and I didn't even read B3 until a couple hours into the test; sadly, this also caused my proof-writing to be... somewhat lacking, so I might end up with three 1s on the three problems I solved on B (darn).
Anyway, these are my solutions, attempting to reconstruct exactly what I wrote on the test. After section A I was very confident since I was pretty sure I had four solid solutions (hopefully this is accurate), so I figured as long as I didn't completely bomb section B I should be in a solid spot to get HM. Unfortunately part A used up all the NT

A1
WLOG assume that A and B lie in the first quadrant. Let
with (WLOG)
; note that
are both positive. Let
. We wish to maximize
over
.
By Shoelace, we have
since
is constant with respect to
, we wish to either maximize or minimize
. Note that
so
(since
is positive). We can easily verify that this gives a larger area than the endpoints
, so this maximizes
.
Let
and
be the projections of
and
onto the
-axis. Then the area of the region enclosed by the hyperbola and
is given by
![$$[B'BPP']-\int_b^{\sqrt{ab}}\frac{1}{x}$$](//latex.artofproblemsolving.com/7/b/7/7b78c46aa5ff60916cbd5f06c487f21c8a43475d.png)


Let
be the projection of
onto the
-axis. Then the area of the region enclosed by the hyperbola and
, similar to above, is given by
![$$[A'APP']-\int_{\sqrt{ab}}^{a}\frac{1}{x}$$](//latex.artofproblemsolving.com/9/7/6/976f079a3d234a78acaaa5c2450fa814b535f541.png)


so it suffices to show
, which is true as desired.




![$[ABP]$](http://latex.artofproblemsolving.com/4/8/8/48884bc091512a29bd03f48e81ba55152b010e59.png)
![$p \in [a,b]$](http://latex.artofproblemsolving.com/7/7/2/772334f666a9afaffefda476a1aa007e5075f423.png)
By Shoelace, we have
![$$[ABP]=\left|\frac{\frac{a}{b}+\frac{b}{p}+\frac{p}{a}-\frac{b}{a}-\frac{p}{b}-\frac{a}{p}}{2}\right|$$](http://latex.artofproblemsolving.com/a/b/2/ab292b460f9af30fe54a7bb2ff7303ffb222d741.png)







![$[ABP]$](http://latex.artofproblemsolving.com/4/8/8/48884bc091512a29bd03f48e81ba55152b010e59.png)
Let






![$$[B'BPP']-\int_b^{\sqrt{ab}}\frac{1}{x}$$](http://latex.artofproblemsolving.com/7/b/7/7b78c46aa5ff60916cbd5f06c487f21c8a43475d.png)







![$$[A'APP']-\int_{\sqrt{ab}}^{a}\frac{1}{x}$$](http://latex.artofproblemsolving.com/9/7/6/976f079a3d234a78acaaa5c2450fa814b535f541.png)




A2
The characteristic polynomial of
is
, which has roots
. Hence, by standard recurrence theory,
for some constants
. Furthermore,
and
, so
. Therefore
for any
.
In particular,
. Note that
, and similarly
. Hence
so
, and
is an odd prime factor of
.










In particular,







A3
Note that
as
are the
th roots of unity for
. Therefore, when
is odd, we have
Similarly, when
is relatively prime to
, the numbers
form a complete residue class modulo
, so
since
.
Fix an
, and let
. Then
where
and
are relatively prime. Thus
from the lemma above, and
so
and so we want
.
Note that
which is equal to
as
appears either in
or
and
is multiplicative.












Fix an






![$$\prod_{b=1}^{2015}\left(1+e^{\frac{2\pi abi}{2015}}\right)=\left[\prod_{b=1}^{\frac{2015}{d}}\left(1+e^{\frac{2\pi\frac{a}{d}bi}{\frac{2015}{d}}}\right)\right]^d=2^d$$](http://latex.artofproblemsolving.com/d/1/3/d13b5a8e9fccc76c8bf6db75879ca3a403d7ef03.png)


Note that






A4
Firstly, note that when
,
is even precisely when
. Thus
suppose now there exists an
such that
. Then we require
to be odd, and since
, this implies
. Therefore
. Suppose there exists an
for which
is odd. Then either
or
is one bigger than
, and is thus even. Hence it is impossible to have three consecutive
for which
is odd.
The minimum potential value of
is thus
since we can get arbitrarily close to
by choosing
for sufficiently small
, the maximum possible
is
.

















The minimum potential value of







Section B was much more stressful; I'd been actively hoping there wouldn't be too much calc, but B1 was some annoying diffeq that I didn't really have any good ideas how to approach (other than the coefficients suspiciously looking like

Eventually I finished B2 first, using an induction that I wasn't (and still aren't) fully confident in, but I definitely solved the problem so hopefully it'll get a 10- grade. Finally, with about half an hour left, I finally came up with the insight as to how B1 would die, so I finished that with about 20 minutes left. Then I looked at B4, which was definitely the easiest problem of the set (maybe I should have looked at this before...), but time pressure made me butcher the computation at the end, and apparently Putnam is very unforgiving towards irrelevant mistakes so that might get a 1 T.T
B1
Define
. Note that
. Furthermore,
, so
has at least 5 distinct real roots, and so
has at least 2 distinct real roots. Since
, this implies that
has at least 2 distinct real roots as desired.







Motivation
So I was playing around a lot with writing
as some function of
and hoping to get
equal to the given
, but I couldn't figure out how to make the connection between
and
work. Things like
and
were getting absolutely nowhere; quite unsurprisingly in fact, since this wouldn't actually say much about
anyway.
Eventually I got frustrated and decided to just write
, differentiate three times, and contrive
to give what I wanted. But then
, which FINALLY explained where the cubic coefficients were coming from; this motivates setting
to get the right coefficients. Then we want
to all be the same; this motivates
, and indeed
, huzzah. Then we just rescale to get the original substitution and everything dies from there.









Eventually I got frustrated and decided to just write







B2
I claim that for any
, exactly one of
and
are in the list of sums. We will prove this by induction. Suppose it holds for all
. We will show it holds for
as well. The base when where
is clear from quick computation, as 6 is in the list of sums.
Let
be the set of triples
such that
are the three smallest remaining numbers at some point in the process. By the inductive hypothesis,
,
, and exactly one of
are in
for all
. This accounts for all triples with entries less than or equal to
, so the next sum can only be equal to
or
. This completes the induction since subsequent sums differ by at least
.
As a corollary of the previous induction,
and
are in
for any
. Therefore,
is in the list of sums for any
. Hence
, and so
is in the list of sums for any
. In particular,
is in the list of sums, so the answer is
.






Let












As a corollary of the previous induction,











B4
So I tried to reconstruct what I wrote on the test, but unfortunately I did the computation correctly this time and I have no idea where I went wrong in-contest T.T
Let
. Since
are the side lengths of a triangle,
are positive integers. Furthermore, for any choice of
that have the same parity, there is a unique triple
of positive integers satisfying the above equations. Therefore
where we set
and
respectively.
Hold
constant. Then we have
Hold
constant. Then we have
Which is equal to

Let








Hold





Better finish
From
we decompose into
which is just a product of easily-computable geometric series. I really don't know how I missed this T.T

